22

Considering an SDE of the form

$$dX_t = \mu(X_t, t)dt + \sigma(X_t, t)dW_t ,$$

where $W_t$ is a Wiener process, is there a set of necessary and sufficient conditions on the structure of the functions $\mu$ and $\sigma$ such that $X_t$ is multivariate normal (found perhaps by solving the Fokker-Planck equation)?

STJ
  • 213
adityar
  • 1,484
  • 2
    InfProbSciX, do you know of functions $\mu(X_t,t)$, $\sigma(t)$ where $X_t$ is not multivariate normal? – Sextus Empiricus Dec 04 '18 at 16:16
  • 1
    @MartijnWeterings I'm not too sure about this but, if $\mu(X_t) = (X_t + a)^{-1}$, for $a$ close to $0$, I probably wouldn't expect $X_t$ to be multivariate normal. I'll do a few simulations now to check. Would it be surprising to find an $X_t$ that's not mvn? – adityar Dec 04 '18 at 16:26
  • 2
    I do not know much about this stuff, and I was actually wondering how such SDE that depends on a Weiner process, would be creating a $X_t$ that is not multivariate normal. I myself imagine that it (not being multivariate normal) can be caused by terms like $X_t dW_t$ (like in a geometric Brownian motion) or anything else that would cause some interaction of the different $dW_t$. – Sextus Empiricus Dec 04 '18 at 16:38
  • 1
    That's an interesting comment and I think you're right. $\mu = 0, \sigma(X_t) = 1/X_t$ generates some weird looking (multivariate) distributions that aren't normal. I think that a lot of GPs can't actually be represented as SDEs and vice versa so I'm really interested in where they intersect. The OU process is an obvious example, but it's too restrictive, and on the other hand both GPs and state space models are incredibly powerful. – adityar Dec 04 '18 at 16:47
  • Looking back at this - there's a relationship between some stationary gaussian processes and state space models that has been shown by Simo Särkkä and Arno Solin. An example is shown in this slide deck titled State Space Representation of Gaussian Processes. – adityar Jan 08 '21 at 20:28
  • 1
    My guess would be iff the diffusion term depends on X Eg lognormal, vs ornstein uhlenbeck – seanv507 Mar 29 '23 at 11:46
  • If this condition holds, then there must be functions $a(u)$ and $b(u)$ such that $$\frac{d}{dw}P\big[X_2=w|X_0=u\big]=P\bigX_2=w|X_0=u\big$$ where the probability satisfies $$P\big[X_2=w|X_0=u\big]=\int P\big[X_2=w|X_1=v\big] P\big[X_1=v|X_0=u\big]dv$$ and leads to an integral equation that someone might be able to solve. – Matt F. Jun 14 '23 at 18:50

2 Answers2

5

First, (to address some of the comments under the main post) it should be noted that we can easily formulate an SDE whose solution is not Gaussian at fixed times. A famous example is the geometric Brownian,

$$ X_t = \mu X_t dt + \sigma X_t dB_t $$

The solution to which is,

$$ X_t = X_0 \exp((\mu - \sigma^2/2)t + \sigma B_t) $$

as we can verify by a simple application of Ito's lemma. Letting $f(t, x) = \exp((\mu - \sigma^2/2)t + \sigma x)$

\begin{align*} dX_t &= f_t(t, B_t)dt + f_x(t, B_t) dB_t + 1/2f_{xx}(t, B_t)d[B_t,B_t] \\ &= ((\mu - \sigma^2/2)t dt + \sigma dB_t + \sigma^2/2 dt)X_t \\ &= \mu X_t dt + \sigma X_t dB_t \end{align*}

It is clear that, fixing $t$, we have $\log(X_t) \sim N( \log(X_0) + (\mu - \sigma^2/2)t, \sigma^2 t)$. So $X_t$ is not a GP.

Now, with that said, I don't know of a necessary condition of the type you are after. However a widely-known sufficient condition is that $\mu$ and $\sigma$ be constant in $X_t$. For details on a proof see this post, https://math.stackexchange.com/questions/4036081/why-is-int-0t-gsdb-s-sim-mathcaln-left0-int-0t-g2s-ds-right.

kibble
  • 61
  • Hi, sorry I don't know much about stochastic calculus. How does your $dB_t$ relate to OP's $dW_t$? – John Madden Jun 15 '23 at 18:48
  • 1
    Ah apologies they are both standard ways to represent a Brownian motion. Wiener process is another name for a Brownian. – kibble Jun 16 '23 at 00:27
4

TLDR

We can assume a Gaussian as solution and insert that into the Fokker Planck equations, to get the potential forms of $\mu$ and $\sigma$ that have the Gaussian as solution.


Let's describe the potential solutions as a Gaussian distribution with a mean $M(t)$ and variance $V(t)$ that can be time dependent:

$$g=g(x,t) = \frac{1}{\sqrt{2\pi V(t)}} e^{-\frac{(x-m(t))^2}{2V(t)}}$$

Where I write $g$ as short for $g(x,t)$

The Fokker Planck equation will have a time and place dependent diffusion term and an external force.

$$\begin{array}{rcl} g_t &=& - \overbrace{ \left[ \mu g\right]_{x}}^{\text{force term}} + \overbrace{ \left[Dg\right]_{xx}}^{\text{diffusion term}} \\ &=& g_x\mu + g\mu_x + D_{xx}g + 2 D_xg_x + Dg_{xx} \end{array} $$

where we use subscripts for partial derivatives and $\mu = \mu(x,t)$ and $D=D(x,t)$ are considered functions of time and space.

the partial derivatives of the Gaussian density $g$ (with potentially time varying mean and variance) are

$$\begin{array}{rcl} g_t/g &=& M_t \frac{x-M}{V} + V_t\frac{(x-M)^2-V}{2V^2} \\ g_{x}/g &=& - \frac{x-M}{V} \\ g_{xx}/g&=& - \frac{1}{V} +\left( \frac{x-M}{V}\right)^2 \\ \end{array}$$

which we can fill into the Fokker Planck equation

$$M_t(x-M) + 0.5 V_t\frac{(x-M)^2-V}{V} = (x-M) \mu + V (-\mu_x + D_{xx}) - 2 D_x (x-M) + D\frac{(x-M)^2-V}{V}$$

and after some rearrangement

$$0 = (x-M) (\mu-M_t) + V (-\mu_x + D_{xx}) - 2 D_x (x-M) + (D-0.5V_t) \frac{(x-M)^2-V}{V}$$

The time variations of variance $V_t$ and mean $M_t$ can be absorbed into the diffusion $D$ and drift $\mu$, and we can treat them as equal to zero without loss of generality. Also we can apply a transformation and scaling of the $x$ coordinate such that $M = 0$ and $V=1$.

This simplifies the equation to a stationary solution with $M=1$ and $V=1$

$$x\mu - \mu_x = D_{xx} - 2 x D_x + D (x^2-1)$$


At this time it might be better to go back to using the original equation

$$\left[Dg\right]_{xx} - \left[\mu g\right]_{x}= \left[(-xD+D_x-\mu)g\right]_{x} = 0$$

Note that if $h = c e^{x^2/2}$ then $\left[hg\right]_{x} = 0$

So for stationary solutions with zero mean and variance one we have the condition

$$\mu = -xD+D_x + c e^{x^2/2}$$

and we can get other solutions by scaling and translation of the $\mu$ and $D$ that satisfy this condition.

This condition is a necessary condition but not a sufficient condition. When we use $c \neq 0$ in the example below then we do not get a stationary solution. The reason is because it represents an unrealistic case where a constant flux is added. If the flux is constant everywhere then the change in time is zero. However, what does this constant flux mean? It means that at infinity there must be a (unrealistic) non-zero flux as well and the force term is blowing up exponentially in order to achieve this.

Simulations with a funny case

For example let's take $D = 2+x^2$ then $\mu = c \cdot e^{x^2/2} - x^3$

Then we could simulate the distribution with a random walk and it looks as following:

example with a funny function set.seed(1) xs = seq(-5,5, 0.01)

drift = function(x) {
  0*exp(x^2/2) - x^3 + 0*x
}

Diff = function(x) { 2+x^2 }

n = 10^6 x = rep(NA,n)

x[1] = 0 dt = 0.01 for (i in 2:n) { b = rnorm(1,0,sqrt(dt)) x[i] = x[i-1] + drift(x[i-1])dt + sqrt(2Diff(x[i-1]))*b }

hist(x, breaks = seq(-40,40,0.2), freq = 0, xlim = c(-5,5)) lines(xs,dnorm(xs))

  • related:https://stats.stackexchange.com/questions/465958/ – Sextus Empiricus Jun 16 '23 at 15:25
  • An option can be to focus on the time homogeneous case where both $\mu$ and $\sigma$ do not depend on $t$. If a stationary density say $\psi$ exists then we have $\partial^2_{x^2}[\sigma^2 \psi] - 2 \partial_x [\mu \psi] = 0$, see Karlin & Taylor A Second Course in Stochastic Processes Vol. 2 formula (5.31). By choosing functions $\mu(x)$ and $\sigma^2(x)$ satisfying this condition with $\psi$ being a normal density, one may find "exotic" Gaussian diffusions out of the case where $\mu$ is linear and $\sigma^2$ is constant. – Yves Jun 16 '23 at 15:55
  • Yes your formula does not seem consistent with that given in K&T which I believe is a very solid reference. – Yves Jun 16 '23 at 17:23
  • This is awesome! – adityar Jan 24 '24 at 11:38
  • @adityar it was a while ago that I created this post. I notice now a mistake with the last term in $\mu = -xD+D_x + c e^{x^2/2}$. In the simulations I used $c=0$ and I could not make it work for non-zero cases. I thought it was because of issues with computations like rounding of. Now I see that it is because the term corresponds to an additional flux that is non-zero and constant in the entire space. This means that we need to be moving an infinite amount of probability mass at infinity. – Sextus Empiricus Jan 25 '24 at 20:02